Những câu hỏi liên quan
Thị Thanh Thảo Tô
Xem chi tiết
Akai Haruma
11 tháng 8 2017 lúc 3:34

Lời giải:

Sử dụng công thức \(\log_ab=\frac{\ln b}{\ln a}\)

\(\Rightarrow A=\frac{\ln 2}{\ln 3}.\frac{\ln 3}{\ln 4}.\frac{\ln 4}{\ln 5}....\frac{\ln 15}{\ln 16}\)

\(\Leftrightarrow A=\frac{\ln 2}{\ln 16}=\log_{16}2=\frac{1}{4}\)

Đáp án C.

Minh Anh
Xem chi tiết
Buddy
Xem chi tiết
Hà Quang Minh
24 tháng 8 2023 lúc 0:46

\(a,A=log_23\cdot log_34\cdot log_45\cdot log_56\cdot log_67\cdot log_78\\ =log_28\\ =log_22^3\\ =3\\ b,B=log_22\cdot log_24...log_22^n\\ =log_22\cdot log_22^2...log_22^n\\ =1\cdot2\cdot...\cdot n\\ =n!\)

tuấn giã văn
Xem chi tiết
Akai Haruma
26 tháng 11 2017 lúc 22:27

Lời giải:

Đặt \(\log_9a=\log_{12}b=\log_{16}(a+b)=t\)

\(\left\{\begin{matrix} a=9^t\\ b=12^t\\ a+b=16^t\end{matrix}\right.\Rightarrow 9^t+12^t=16^t\)

Chia 2 vế cho \(12^t\) ta có:

\(\left(\frac{9}{12}\right)^t+1=\left(\frac{16}{12}\right)^t\)

\(\Leftrightarrow \left(\frac{3}{4}\right)^t+1=\left(\frac{4}{3}\right)^t\) (1)

Đặt \(\frac{a}{b}=\left(\frac{9}{12}\right)^t=\left(\frac{3}{4}\right)^t=k\). Thay vào (1):

\(k+1=\frac{1}{k}\Leftrightarrow k^2+k-1=0\)

\(\Leftrightarrow \frac{a}{b}=k=\frac{-1+ \sqrt{5}}{2}\) (do \(k>0\) nên loại TH \(k=\frac{-1-\sqrt{5}}{2}\) )

Thấy \(\frac{-1+\sqrt{5}}{2}\in (0;\frac{2}{3})\) nên chọn đáp án b

Nghi Quỳnh
Xem chi tiết
Nguyễn Lê Phước Thịnh
18 tháng 5 2022 lúc 8:11

Chọn B

Buddy
Xem chi tiết
datcoder
15 tháng 8 2023 lúc 15:08

a) \(\log_a\left(a^2b\right)=\log_aa^2+\log_ab=2.\log_aa+\log_ab=2.1+2=4\)

b) \(\log_a\dfrac{a\sqrt{a}}{b\sqrt[3]{a}}=\log_a\left(a\sqrt{a}\right)-\log_a\left(b\sqrt[3]{b}\right)=\log_aa^{\dfrac{3}{2}}-\log_ab^{\dfrac{4}{3}}=\dfrac{3}{2}.\log_aa-\dfrac{4}{3}\log_ab=\dfrac{3}{2}.1-\dfrac{4}{3}.2=-\dfrac{7}{6}\)

c) \(\log_a\left(2b\right)+\log_a\left(\dfrac{b^2}{2}\right)=\log_a2+\log_ab+\log_ab^2-\log_a2=\log_ab+2\log_ab=3\log_ab=3.2=6\)

Nguyễn Lê Phước Thịnh
14 tháng 8 2023 lúc 13:52

a: \(=log_aa^2+log_ab=2+2=4\)

b: \(log_a\left(\dfrac{a\sqrt{a}}{b\sqrt[3]{b}}\right)=log_aa^{\dfrac{3}{2}}-log_ab^{\dfrac{4}{3}}\)

=3/2-4/3*2

=3/2-8/3

=9/6-16/6=-7/6

c: \(log_a\left(2b\right)+log_a\left(\dfrac{b^2}{2}\right)\)

\(=log_a\left(2b\cdot\dfrac{b^2}{2}\right)=log_a\left(b^3\right)=3\cdot2=6\)

Buddy
Xem chi tiết
Hà Quang Minh
22 tháng 9 2023 lúc 14:42

\({a^{\frac{1}{2}}} = b \Leftrightarrow {\log _a}b = \frac{1}{2} \Leftrightarrow 2{\log _a}b = 1\)

Chọn B.

Như Quỳnh
Xem chi tiết
Akai Haruma
12 tháng 11 2018 lúc 19:58

Bài 1:

\(A=\log_380=\log_3(2^4.5)=\log_3(2^4)+\log_3(5)\)

\(=4\log_32+\log_35=4a+b\)

\(B=\log_3(37,5)=\log_3(2^{-1}.75)=\log_3(2^{-1}.3.5^2)\)

\(=\log_3(2^{-1})+\log_33+\log_3(5^2)=-\log_32+1+2\log_35\)

\(=-a+1+2b\)

Akai Haruma
12 tháng 11 2018 lúc 20:05

Bài 2:

\(\log_{30}8=\frac{\log 8}{\log 30}=\frac{\log (2^3)}{\log (10.3)}=\frac{3\log2}{\log 10+\log 3}\)

\(=\frac{3\log (\frac{10}{5})}{1+\log 3}=\frac{3(\log 10-\log 5)}{1+\log 3}=\frac{3(1-b)}{1+a}\)

Akai Haruma
13 tháng 11 2018 lúc 8:35

Bài 3:

\(\log_{27}5=a; \log_87=b; \log_23=c\)

\(\Leftrightarrow \frac{\ln 5}{\ln 27}=a; \frac{\ln 7}{\ln 8}=b; \frac{\ln 3}{\ln 2}=c\)

\(\Leftrightarrow \frac{\ln 5}{\ln (3^3)}=a; \frac{\ln 7}{\ln (2^3)}=b; \ln 3=c\ln 2\)

\(\Leftrightarrow \frac{\ln 5}{3\ln 3}=a; \frac{\ln 7}{3\ln 2}=b; \ln 3=c\ln 2\)

\(\Rightarrow \frac{\ln 5}{3c\ln 2}=a; \frac{\ln 7}{3\ln 2}=b\)

\(\Rightarrow \ln 35=\ln 5+\ln 7=3ac\ln 2+3b\ln 2\)

Do đó:
\(D=\log_6 35=\frac{\ln 35}{\ln 6}=\frac{\ln 35}{\ln 2+\ln 3}=\frac{\ln 35}{\ln 2+c\ln 2}=\frac{3ac\ln 2+3b\ln 2}{\ln 2+c\ln 2}\)

\(=\frac{3ac+3b}{1+c}\)

Buddy
Xem chi tiết
Nguyễn Lê Phước Thịnh
15 tháng 8 2023 lúc 19:59

\(log_a\left(a^3b^2\right)=log_aa^3+log_ab^2=3+2\cdot log_ab\)

=>B

Buddy
Xem chi tiết
Hà Quang Minh
24 tháng 8 2023 lúc 9:12

\(\dfrac{a^2\cdot\sqrt[3]{a}\cdot\sqrt[5]{a^4}}{\sqrt[4]{a}}=\dfrac{a^2\cdot a^{\dfrac{1}{3}}\cdot a^{\dfrac{4}{5}}}{a^{\dfrac{1}{4}}}=\dfrac{a^{\dfrac{47}{15}}}{a^{\dfrac{1}{4}}}=a^{\dfrac{173}{60}}\)

\(\Rightarrow log_a\left(\dfrac{a^2\cdot\sqrt[3]{a}\cdot\sqrt[5]{a^4}}{\sqrt[4]{a}}\right)=log_a\left(a^{\dfrac{173}{60}}\right)=\dfrac{173}{60}\)

\(a^{2log_a\left(\dfrac{\sqrt{105}}{30}\right)}=a^{log_a\left(\dfrac{7}{60}\right)}=\dfrac{7}{60}\)

Vậy \(B=\dfrac{173}{60}+\dfrac{7}{60}=\dfrac{180}{60}=3\)